After College
by djmathman, May 23, 2016, 2:07 AM
home life seems so boring in comparison >.<
I've been thinking a bit about the whole college application process as of late. Don't get me wrong, it's not that I don't like CMU - I'm extremely happy where I am right now, and I definitely feel that I made the right choice. But sometimes, it's interesting to reflect on how my life might be different had my thought process been different.
I remember once during my AMA when I was asked about why I chose CMU one of the reasons I gave was the following:
But now that I think about it, I'm not entirely sure that I would have gone to MIT even if I had gotten in. After all, there is a sort comfort that comes from being at a place where the amount of sheer brainpower floating all over the place isn't super overwhelming. At CMU, there's enough interest in mathematics that I feel comfortable and sociable with others but not so much that I'm constantly facing imposter syndrome, and that sort of balance feels really nice. (Of course, had I gotten in, it probably would have been because I would have made USAMO the previous year, and so maybe I'd be slightly better at math than I am now. Oh well.)
Easy
Solution
Solution
Also darn 2018 AMC/AIME submissions are due by the end of the month, and I don't feel like I have enough good AIME proposals to make up for all the times I pestered the MAA to let me write problems for them D:
I've been thinking a bit about the whole college application process as of late. Don't get me wrong, it's not that I don't like CMU - I'm extremely happy where I am right now, and I definitely feel that I made the right choice. But sometimes, it's interesting to reflect on how my life might be different had my thought process been different.
I remember once during my AMA when I was asked about why I chose CMU one of the reasons I gave was the following:
djmathman wrote:
I didn't get into MIT. (This wasn't as much of an issue as I grew older, but even in 11th grade I was dead set on getting into MIT and going there, despite what anybody else said about it.)
IMC 2010.1 wrote:
Let
. Prove that ![\[\int_a^b (x^2+1)e^{-x^2} dx \geq e^{-a^2} - e^{-b^2}.\]](//latex.artofproblemsolving.com/4/b/8/4b81207bed4a768652c4373d94ae1f1aa4be748c.png)

![\[\int_a^b (x^2+1)e^{-x^2} dx \geq e^{-a^2} - e^{-b^2}.\]](http://latex.artofproblemsolving.com/4/b/8/4b81207bed4a768652c4373d94ae1f1aa4be748c.png)
Easy
Note that
as desired.

IMC 2010.2 wrote:
Compute the sum of the series ![\[\sum_{k=0}^{\infty} \frac{1}{(4k+1)(4k+2)(4k+3)(4k+4)} = \frac{1}{1\cdot2\cdot3\cdot4} + \frac{1}{5\cdot6\cdot7\cdot8} + \cdots.\]](//latex.artofproblemsolving.com/5/6/0/5602abbee969b9d697fbf42a691ad47882539927.png)
![\[\sum_{k=0}^{\infty} \frac{1}{(4k+1)(4k+2)(4k+3)(4k+4)} = \frac{1}{1\cdot2\cdot3\cdot4} + \frac{1}{5\cdot6\cdot7\cdot8} + \cdots.\]](http://latex.artofproblemsolving.com/5/6/0/5602abbee969b9d697fbf42a691ad47882539927.png)
Solution
First note that
We can use a similar reduction process a few more times to obtain the full simplification
It thus remains to compute the sum
and then divide the result by
to obtain our final result.
For all
for which the function is defined, let
Observe that
In an attempt to integrate this, note that for all
,
Each of these terms is easily integrable, and we obtain
Note that although
is not defined for
, its limit as
approaches
exists, and so by continuity this expression for
is also valid for
. Hence
and so the desired sum is 

![\[\frac{1}{(4k+1)(4k+2)(4k+3)(4k+4)} = \dfrac16\left(\dfrac{1}{4k+1}-\dfrac{3}{4k+2}+\dfrac{3}{4k+3} - \dfrac{1}{4k+4}\right).\]](http://latex.artofproblemsolving.com/8/c/5/8c50d507c953b8c1a6631eb4e5b9b38c99409a7e.png)
![\[\sum_{k=0}^\infty \left(\dfrac{1}{4k+1}-\dfrac{3}{4k+2}+\dfrac{3}{4k+3} - \dfrac{1}{4k+4}\right)\]](http://latex.artofproblemsolving.com/7/d/3/7d363779773b1ab21e13c5c5809390e35497d521.png)

For all

![\[F(x) = \sum_{k=0}^\infty\left(\dfrac{x^{4k+1}}{4k+1}-\dfrac{3x^{4k+2}}{4k+2}+\dfrac{3x^{4k+3}}{4k+3} - \dfrac{x^{4k+4}}{4k+4}\right).\]](http://latex.artofproblemsolving.com/b/f/9/bf9d49a99d02e3aaa40ed87b4645c60debe69afa.png)


![\[\dfrac{(1-x)^3}{1-x^4} = \dfrac{(1-x)^2}{(x+1)(x^2+1)} = \dfrac{2}{x+1} - \dfrac{x}{x^2+1} - \dfrac{1}{x^2+1}.\]](http://latex.artofproblemsolving.com/8/4/3/843931d565c9b65c1f42258c7e46613e0031f322.png)
![\[F(x) = 2\ln(x+1) - \dfrac12\ln(x^2+1) - \arctan x.\]](http://latex.artofproblemsolving.com/d/3/c/d3c495bfa735f4f28f947b008de764c025fac069.png)






![\[F(1) = 2\ln 2 - \frac12\ln 2 - \arctan 1 = \dfrac32\ln 2 - \dfrac{\pi}4\]](http://latex.artofproblemsolving.com/6/7/4/674c667cc0ebe28e267e3923fd1d9671048f4076.png)

IMC 2010.3 wrote:
Define the sequence
inductively by
and
for each
. Compute ![\[\lim_{n \to \infty} \frac{x_1 \cdot x_2 \cdot x_3 \cdot ... \cdot x_n}{x_{n+1}}.\]](//latex.artofproblemsolving.com/4/d/0/4d03bf968b6c215e086bc9f985844ea7299968a2.png)




![\[\lim_{n \to \infty} \frac{x_1 \cdot x_2 \cdot x_3 \cdot ... \cdot x_n}{x_{n+1}}.\]](http://latex.artofproblemsolving.com/4/d/0/4d03bf968b6c215e086bc9f985844ea7299968a2.png)
Solution
So it turns out this problem is a very quick solve once you discover the right pattern...
LEMMA: For all
, we have
Proof. We proceed by induction. The base case of
is trivial. For the inductive step, assume the result holds true for some
. Then
as desired. 
From here the result is immediate, as![\[\lim_{n \to \infty} \frac{x_1 \cdot x_2 \cdot x_3 \cdot ... \cdot x_n}{x_{n+1}} = \lim_{n\to\infty}\sqrt{\frac{x_{n+1}^2-4}{x_{n+1}^2}} = \sqrt{\lim_{n\to\infty}\left(1 - \dfrac{4}{x_{n+1}^2}\right)} = \boxed{1}.\]](//latex.artofproblemsolving.com/9/b/e/9bea6d9bc3d37e581630005dd33b66e224e91d20.png)
LEMMA: For all

![\[\dfrac{x_1x_2\cdots x_n}{x_{n+1}} = \sqrt{\dfrac{x_{n+1}^2-4}{x_{n+1}^2}}.\]](http://latex.artofproblemsolving.com/6/3/7/637dd58fb3c2ba3de7a74070f329c1f79a4eeff8.png)




From here the result is immediate, as
![\[\lim_{n \to \infty} \frac{x_1 \cdot x_2 \cdot x_3 \cdot ... \cdot x_n}{x_{n+1}} = \lim_{n\to\infty}\sqrt{\frac{x_{n+1}^2-4}{x_{n+1}^2}} = \sqrt{\lim_{n\to\infty}\left(1 - \dfrac{4}{x_{n+1}^2}\right)} = \boxed{1}.\]](http://latex.artofproblemsolving.com/9/b/e/9bea6d9bc3d37e581630005dd33b66e224e91d20.png)
Also darn 2018 AMC/AIME submissions are due by the end of the month, and I don't feel like I have enough good AIME proposals to make up for all the times I pestered the MAA to let me write problems for them D:
This post has been edited 3 times. Last edited by djmathman, May 23, 2016, 2:19 AM